PT70.S1.Q9 - Astronomer: Proponents of the hypothesis that

MambaMentality93MambaMentality93 Live Member
edited March 2019 in Logical Reasoning 503 karma

I don't understand how A can be the answer.
A says "no evidence in favor of a hypothesis," but the proponents of the hypothesis is giving us evidence. He explains the fossil thing etc how come this doesn't count as evidence backing up his claim?

Admin note: edited title
https://7sage.com/lsat_explanations/lsat-70-section-1-question-09/

Comments

  • BlindReviewerBlindReviewer Alum Member
    855 karma

    I've seen this question a couple times, but I still have difficulty with answer choice E haha will try to talk my way through it here.

    Before that, though, A is right because this stimulus is giving us a pretty cookie-cutter flaw, which sometimes manifests itself in the answer choice, "Dismisses a conclusion/hypothesis merely on the basis that an inadequate argument is given for it." An easy to understand analogous argument is "God doesn't exist because Herman knows everything and he told me God doesn't exist." Sure that argument is absolute garbage, but you can't conclude from this argument that God does in fact exist -- you can only conclude that it's a bad argument.

    Similarly here, the Astronomer is saying that there's a good chance the proponent's hypothesis is false because their argument is weak -- they only rule out one alternative explanation and they don't actually provide any support for their own hypothesis. But you can't conclude that just because their argument is bad or that they don't provide evidence that their hypothesis is false -- you can only conclude that the case for their hypothesis is not that strong.

    I have trouble with E because it's so convoluted. I think the main thing with E is that the Astronomer doesn't actually "grant the truth" of the claims -- they just say that these hypotheses are given, and it's not enough. I don't think it's taken for granted that these claims are definitely true. Also I think the second part "does nothing to strengthen the astronomer's argument" is false -- I think bringing up the claims does, however partially, strengthen the argument because it's true that the proponents' argument isn't great. It strengthens, but not enough to conclude that the proponents' hypothesis is "false."

Sign In or Register to comment.